The Student Room Group

OCR (Not MEI) Core 2 Discussion Thread 17th May 2013 + Jan 13 Paper and MS

Scroll to see replies

Reply 60
I tried to do the tangent area under the graph question 2 ways...

First I got 37/30 by subtracting the area of the right angled triangle.

Then, to check, i tried integrating. (x^3/2 - 1) - (3x-5) and got a completely different answer of 1.9

Surely both methods are valid and should both produce the exact same answer??

In the end I stuck with the original 37/30, the rest of the paper was pretty fair.


Posted from TSR Mobile
Reply 61
Original post by purplemind
I got -11.


Me too.
Reply 62
Original post by mia_hilton
but how did you know the coordinate of the tangent when it hit the x axis?


The equation of the tangent was y=3x-5, so when y=0, 3x=5, x=5/3 so coordinates are (5/3, 0)


Posted from TSR Mobile
Original post by _JC95
I tried to do the tangent area under the graph question 2 ways...

First I got 37/30 by subtracting the area of the right angled triangle.

Then, to check, i tried integrating. (x^3/2 - 1) - (3x-5) and got a completely different answer of 1.9

Surely both methods are valid and should both produce the exact same answer??

In the end I stuck with the original 37/30, the rest of the paper was pretty fair.


Posted from TSR Mobile


Yeah 37/30 is the right answer!
If you integrated then you had to use the coord where the tangent crosses the x axis which was 5/3
Therefore you integrate between 5/3 and 1
You used 4 and 1 which is why you got 1.9 because I did that too but realised..
Reply 65
Original post by _JC95
The equation of the tangent was y=3x-5, so when y=0, 3x=5, x=5/3 so coordinates are (5/3, 0)


Posted from TSR Mobile


thanks it makes so much sense........wishing i could turn back time right now :frown:
Reply 66
anyone want to hazard a guess at grade boundaries?

Could you just quote a value of a and b because that is what it said?
Reply 68
That log question was horrifichorrific
Reply 69
Okay that was horrible! At least most people found it hard so hopefully low grade boundaries! What was going on with the big log question!? Had no clue!
Original post by stefanconstant
Could you just quote a value of a and b because that is what it said?


Yeah it asked for possible values for a and b, so as long as your a was >1 and 0<b<1
Reply 71
Oh nooooo. I put 1.9. Are you sure it's not 1.9???

Posted from TSR Mobile
Original post by niaphonic
Yeah it asked for possible values for a and b, so as long as your a was >1 and 0<b<1

Sweet I said a=2 and b= 1/2
Original post by stefanconstant
Sweet I said a=2 and b= 1/2


Should be fine, I put a=1.5 b=0.5 too. I just tried out random values that looked roughly right, brain wasn't working... nearly wrote b= -1
Reply 74
Original post by stefanconstant
Yeah 37/30 is the right answer!
If you integrated then you had to use the coord where the tangent crosses the x axis which was 5/3
Therefore you integrate between 5/3 and 1
You used 4 and 1 which is why you got 1.9 because I did that too but realised..


Yeah but in dying minutes i also did it with 1 and 5/3 and did not get the same answer either...


Posted from TSR Mobile
Original post by Cowcat
Okay that was horrible! At least most people found it hard so hopefully low grade boundaries! What was going on with the big log question!? Had no clue!


It threw me the first time so I left it to the end and tried again, managed it the second time! The question showed you had to use log to base 2, and then had to use ab=2 to simplify the expression at the end (ie. replace b with 2/a). I forgot both of those the first time I tried to do it..
Original post by niaphonic
Should be fine, I put a=1.5 b=0.5 too. I just tried out random values that looked roughly right, brain wasn't working... nearly wrote b= -1

Haha yeah, thanks!
Original post by _JC95
Yeah but in dying minutes i also did it with 1 and 5/3 and did not get the same answer either...


Posted from TSR Mobile

Well 37/30
is the right answer so you're fine..
Did you integrate it right?
Reply 78
Original post by _JC95
I tried to do the tangent area under the graph question 2 ways...

First I got 37/30 by subtracting the area of the right angled triangle.

Then, to check, i tried integrating. (x^3/2 - 1) - (3x-5) and got a completely different answer of 1.9

Surely both methods are valid and should both produce the exact same answer??

In the end I stuck with the original 37/30, the rest of the paper was pretty fair.


Posted from TSR Mobile



I'm on the same boat as you,

the rest of the paper was fine,
except when i wanted to make sure my answer was right for that question,
i got different answers :/
Reply 79
How many marks would I get for putting 1.9 for that graph question?

Posted from TSR Mobile

Quick Reply

Latest

Trending

Trending